Solving Eigenvalues of Hessian Matrix

Click For Summary
The discussion focuses on finding the eigenvalues of a Hessian matrix derived from the function g(x,y) = x^3 - 3x^2 + 5xy - 7y^2. The Hessian matrix is initially miscalculated, leading to confusion in solving the characteristic equation for eigenvalues. After correcting a numerical error in the Hessian, participants clarify that eigenvalues can be derived as functions of x by solving the quadratic equation. There is also uncertainty about evaluating the Hessian at the point (0,0). The conversation emphasizes the importance of accurate calculations and understanding the process of finding eigenvalues in matrix analysis.
Firepanda
Messages
425
Reaction score
0
g(x,y) = x^3 - 3x^2 + 5xy -7y^2

Hessian Matrix =

6x-6******5

5********-7

Now I have to find the eigenvalues of this matrix, so I end up with the equation (where a = lambda)

(6x - 6 - a)(-7 - a) - 25 = 0

Multiplying out I get:

a^2 - 6xa + 13a - 42x + 17 = 0

How am I supposed to solve for a? Usually I just use the quadratic formula for my eigenvalues..

Should I take a third row/ column for my hessian matrix?
 
Physics news on Phys.org
First off, you have a numerical error in your Hessian. Double check your derivation.

To find the eigenvalues, just solve for a as you would normally (i.e., the eigenvalues will be functions of x).
 
http://img229.imageshack.us/img229/2286/hesssn5.jpg

That's the whole question, I assume I'm doing something wrong because my roots of the quadratic for lamba is very complicated.

And yes thankyou I changed my -7 value in my hessian for -14
 
Last edited by a moderator:
So, what is H(0,0)?
 
D H said:
So, what is H(0,0)?

I've no idea :D this is the 1st hessian question I've ever done, i also previosuly made a thread in the same section about what this notation meant, because I'm not too sure.

Do I just plug in x= 0 and y=0?

So my hessian would be:

-6*****5

5*****-14
 
Question: A clock's minute hand has length 4 and its hour hand has length 3. What is the distance between the tips at the moment when it is increasing most rapidly?(Putnam Exam Question) Answer: Making assumption that both the hands moves at constant angular velocities, the answer is ## \sqrt{7} .## But don't you think this assumption is somewhat doubtful and wrong?

Similar threads

Replies
4
Views
2K
  • · Replies 3 ·
Replies
3
Views
2K
Replies
2
Views
2K
  • · Replies 7 ·
Replies
7
Views
3K
Replies
8
Views
3K
  • · Replies 8 ·
Replies
8
Views
7K
  • · Replies 8 ·
Replies
8
Views
4K
  • · Replies 19 ·
Replies
19
Views
4K
  • · Replies 2 ·
Replies
2
Views
2K
  • · Replies 6 ·
Replies
6
Views
9K